Magnetic field created by a current carrying wire

In summary: Sure, no problem. It is the ends. At one end you have current flowing out of nowhere and at the other end you have current flowing into nowhere. That appearing and disappearing current is the problem since current needs to either travel in loops or start/end in a place where charge is conserved.OK, so the two methods give different results because the current at the ends of the wire is not looping. If the current at the ends of the wire were looping, then the two methods would give the same result.
  • #1
Istvan01
17
0
Hi,

I studied the Maxwell laws and the Biot Savart law and I found something I cannot answer.

If you have a finite wire carrying current (let say 5m long) and you want to determine the magnetic induction vector due to it at some point that has distance r from the wire, you have 2 options I think.

If you use the Biot Savart law and you integrate over the length of the wire you get an answer which is definitely less than the answer you would get if the wire would be infinite.(B=µ0*I/(2*r*
9be4ba0bb8df3af72e90a0535fabcc17431e540a
), where B is the magnetic induction vector, I is the current of the wire and r is the distance from the wire)

However if you use Ampere's law for a circle around the wire (the wire is at the center of the circle and the plane of the circle is perpendicular to the line of the wire) you get that the magnetic field is B=µ0*I/(2*r*
9be4ba0bb8df3af72e90a0535fabcc17431e540a
) since there is only the current I crossing the circle and there is no change in flux. It is the same as the magnetic field generated by an infinite long wire. So that is definitely bigger than u get from the Biot Savart law.

I can't figure out why the 2 methods don't give the same result so if you could help me out please do so. It would be really appreciated.

Thanks,
Istvan
 
Physics news on Phys.org
  • #2
Istvan01 said:
a finite wire carrying current
A finite wire carrying current violates the conservation of charge.

Istvan01 said:
I can't figure out why the 2 methods don't give the same result
The conservation of charge is built into Maxwell’s equations, so a scenario that violates it will cause problems when you use them to analyze it.
 
  • #3
As far as I know, Ampere is for an infinitely long wire (namely where there is no ##z## component of ##\vec B## anywhere).
 
  • #4
Dale said:
A finite wire carrying current violates the conservation of charge.

The conservation of charge is built into Maxwell’s equations, so a scenario that violates it will cause problems when you use them to analyze it.
Under finite wire I mean like a circuit that is big enough that the wire closing the circuit is far away so its effect to the magnetic field is negligible.
 
Last edited by a moderator:
  • #5
BvU said:
As far as I know, Ampere is for an infinitely long wire (namely where there is no ##z## component of ##\vec B## anywhere).
Nah, Ampere's Law always works. It's just that an infinitely long straight wire is a situation where it's straightforward to apply and find B.
 
  • Like
Likes BvU
  • #6
Istvan01 said:
Under finite wire I mean like a circuit that is big enough that the wire closing the circuit is far away so its effect to the magnetic field is negligible.
OK, that doesn’t change my comment. The Biot Savart law can be applied to situations that violate the conservation of charge. Maxwell’s equations cannot. If you choose to apply both to such a scenario then you are guaranteed to run into problems.
 
  • #7
Istvan01 said:
Under finite wire I mean like a circuit that is big enough that the wire closing the circuit is far away so its effect to the magnetic field is negligible.
"Big enough" is another way of saying "infinitely far away." When you apply Ampere's Law to an infinitely long wire, you're taking advantage of the cylindrical symmetry of the system to argue that the magnitude of the field is constant on a circle centered on the wire. If the rest of the circuit is far enough away so that approximation holds true, you're saying it's infinitely far away.

If you use the Biot-Savart law on an finite length of wire to find the field at a point, you have to assume that the contributions from the rest of the circuit cancel at that point. Even though they cancel at that one point, they won't necessarily cancel at other points on a circle which goes through the point in question, so the result derived using Ampere's Law assuming the field strength is constant on the loop doesn't apply.
 
  • Like
Likes Istvan01
  • #8
Dale said:
OK, that doesn’t change my comment. The Biot Savart law can be applied to situations that violate the conservation of charge. Maxwell’s equations cannot. If you choose to apply both to such a scenario then you are guaranteed to run into problems.
Then I don't see how does it violates the conservation of charge... I'm sorry if it's very obvious but I don't understand
 
  • #9
Istvan01 said:
Then I don't see how does it violates the conservation of charge... I'm sorry if it's very obvious but I don't understand
Sure, no problem. It is the ends. At one end you have current flowing out of nowhere and at the other end you have current flowing into nowhere. That appearing and disappearing current is the problem since current needs to either travel in loops or start/end in a place where charge is decreasing/increasing.
 
  • #10
I was a bit hasty in #3, so perhaps I can try to fix things a bit by referring to amperes law in a higly unrealistic case where you manage to get a linear section of current with charge conservation: the charge just piles up at the top end. Would the time derivative of the E-flux make the difference between infinitely long wire and a finite length section aclculated with Biot-Savart ?
 
  • #11
BvU said:
a higly unrealistic case where you manage to get a linear section of current with charge conservation: the charge just piles up at the top end. Would the time derivative of the E-flux make the difference between infinitely long wire and a finite length section aclculated with Biot-Savart ?
Mostly, yes. Having the charge pile up like that will make Maxwell’s equations work, but at the same time it violates one of the assumptions of Biot Savart. In the end it is more important to not violate Maxwell’s equations, so overall I think it is an improvement even if it doesn’t quite reconcile the two
 
  • Like
Likes vsv86 and BvU
  • #12
I agree with Dale. When applying integral form of Ampere's law remember that the requirement on the surface bounded by the imaginary loop you draw around the current line, the loop that you use to get your magnetic field, is only that this surface is bounded by the loop. So I could deform this surface, without moving bounds until finite current line no longer pierces it. The magnetic field would then actually be zero! Thus you get contradictions even with Ampere's law alone.

More specifically the problem is that in magnetostatics ##\vec{\nabla}\times\vec{B}=\mu_0 \vec{J}## (##\vec{B}## magnetic field, ##\vec{J}## current density), but this implies ##\vec{\nabla}.\vec{J}=0##, yet if your current line is finite, then at the ends the divergence of the current density will not be zero, so you are violating the very law you are trying to use.
 
  • Like
Likes Dale
  • #13
@vsv86 that is very well said. The issue with deforming the surface is quite important and very easily forgotten.
 
  • #14
@Dale sorry, I did not tag you, still getting used to forum :-)
 
  • #15
vsv86 said:
The magnetic field would then actually be zero!
It would if there wasn't a term $${1\over c^2 }{\partial \over \partial t}\int \vec E\cdot\vec {dA}\ , $$but there is.
 
  • #16
BvU said:
It would if there wasn't a term $${1\over c^2 }{\partial \over \partial t}\int \vec E\cdot\vec {dA}\ , $$but there is.

Of course. The initial question talked about the Biot-Savart law which only applies in magneto-statics (one must use Jefimenko's equations or equivalent in electrodynamics), so I talked about magnetostatics. In case of full electrodynamics, one will have the extra term due to displacement current, and there will be no contradiction as long as one stays away from Biot-Savart.
 

Related to Magnetic field created by a current carrying wire

What is a magnetic field?

A magnetic field is a region in space where a magnetic force can be detected. It is created by the movement of electric charges, such as in a current carrying wire.

How is a magnetic field created by a current carrying wire?

When an electric current flows through a wire, it creates a circular magnetic field around the wire. The strength of the magnetic field depends on the amount of current flowing through the wire and the distance from the wire.

What is the direction of the magnetic field around a current carrying wire?

The direction of the magnetic field around a current carrying wire is determined by the right hand rule. If you point your thumb in the direction of the current, your fingers will curl in the direction of the magnetic field lines.

How does the strength of the magnetic field change with distance from the wire?

The strength of the magnetic field decreases as you move further away from the wire. This is because the magnetic field lines spread out as they move away from the wire, resulting in a weaker magnetic field at a greater distance.

Can a current carrying wire have multiple magnetic fields around it?

Yes, a current carrying wire can have multiple magnetic fields around it if there are multiple wires carrying current in close proximity. The magnetic fields will interact with each other and create a more complex pattern.

Similar threads

Replies
12
Views
838
Replies
2
Views
76
Replies
8
Views
719
Replies
1
Views
1K
  • Electromagnetism
6
Replies
198
Views
10K
  • Sticky
  • Electromagnetism
Replies
1
Views
2K
Replies
27
Views
1K
Replies
2
Views
854
  • Electromagnetism
Replies
5
Views
361
  • Electromagnetism
Replies
2
Views
431
Back
Top